Difference between revisions of "AIME 2020(MOCK) Problems"

(Problem 2)
(Problem 2)
Line 6: Line 6:
 
==Problem 2==
 
==Problem 2==
  
Let <math>K</math> be a set of polynomials <math>P(x)</math> such that the roots of <math>P(x)</math>
+
Let <math>K</math> be a set of polynomials <math>P(x)</math> such that the roots of <math>P(x)</math> are <math>cos \frac{\pi}{2}</math>, <math>cos \frac{\pi}{4}</math>

Revision as of 18:25, 21 May 2020

Problem 1

Let $N$ be $112123123412345... (1000 digits)$. What is the remainder when $N$ is divided by $21$?


Problem 2

Let $K$ be a set of polynomials $P(x)$ such that the roots of $P(x)$ are $cos \frac{\pi}{2}$, $cos \frac{\pi}{4}$